Polynomials are irreducible in $mathbb{Z}[x]$ if they are irreducible in $mathbb{Z}/pmathbb{Z}$, why p needs...












2












$begingroup$


The proof says we define the modulo function as the natural homomorphism,
$$ barphi: mathbb Z to mathbb Z / pmathbb Z$$
Then if $h in mathbb Z[x]$ is reducible, the identity
$$ barphi(h) = barphi(fg) = barphi(f) barphi(g) $$
proves $h (text{mod} p)$ is also reducible in $mathbb Z_p$



However, this proof tells nothing about $p$ being prime. And consider
$$ barphi: mathbb Z to mathbb Z / 4mathbb Z $$
is also a homomorphism, defined by
$$ z mapsto z + 4mathbb Z$$










share|cite|improve this question











$endgroup$












  • $begingroup$
    $2x^2+4$ is irreducible over $mathbb{Z}$, but over $mathbb{Z}_4$ it splits as $(2x)x$.
    $endgroup$
    – Randall
    Dec 10 '18 at 13:47










  • $begingroup$
    You're right, there's no need for $p$ to be prime.
    $endgroup$
    – Stephen
    Dec 10 '18 at 13:56










  • $begingroup$
    @Randall That does not mean the statement is wrong, since from the statement, any polynomial reducible in $mathbb Z_4$ says nothing about its irreducibility in $mathbb Z$
    $endgroup$
    – Astrick Harren
    Dec 10 '18 at 13:57










  • $begingroup$
    @AstrickHarren good point
    $endgroup$
    – Randall
    Dec 10 '18 at 14:05
















2












$begingroup$


The proof says we define the modulo function as the natural homomorphism,
$$ barphi: mathbb Z to mathbb Z / pmathbb Z$$
Then if $h in mathbb Z[x]$ is reducible, the identity
$$ barphi(h) = barphi(fg) = barphi(f) barphi(g) $$
proves $h (text{mod} p)$ is also reducible in $mathbb Z_p$



However, this proof tells nothing about $p$ being prime. And consider
$$ barphi: mathbb Z to mathbb Z / 4mathbb Z $$
is also a homomorphism, defined by
$$ z mapsto z + 4mathbb Z$$










share|cite|improve this question











$endgroup$












  • $begingroup$
    $2x^2+4$ is irreducible over $mathbb{Z}$, but over $mathbb{Z}_4$ it splits as $(2x)x$.
    $endgroup$
    – Randall
    Dec 10 '18 at 13:47










  • $begingroup$
    You're right, there's no need for $p$ to be prime.
    $endgroup$
    – Stephen
    Dec 10 '18 at 13:56










  • $begingroup$
    @Randall That does not mean the statement is wrong, since from the statement, any polynomial reducible in $mathbb Z_4$ says nothing about its irreducibility in $mathbb Z$
    $endgroup$
    – Astrick Harren
    Dec 10 '18 at 13:57










  • $begingroup$
    @AstrickHarren good point
    $endgroup$
    – Randall
    Dec 10 '18 at 14:05














2












2








2


0



$begingroup$


The proof says we define the modulo function as the natural homomorphism,
$$ barphi: mathbb Z to mathbb Z / pmathbb Z$$
Then if $h in mathbb Z[x]$ is reducible, the identity
$$ barphi(h) = barphi(fg) = barphi(f) barphi(g) $$
proves $h (text{mod} p)$ is also reducible in $mathbb Z_p$



However, this proof tells nothing about $p$ being prime. And consider
$$ barphi: mathbb Z to mathbb Z / 4mathbb Z $$
is also a homomorphism, defined by
$$ z mapsto z + 4mathbb Z$$










share|cite|improve this question











$endgroup$




The proof says we define the modulo function as the natural homomorphism,
$$ barphi: mathbb Z to mathbb Z / pmathbb Z$$
Then if $h in mathbb Z[x]$ is reducible, the identity
$$ barphi(h) = barphi(fg) = barphi(f) barphi(g) $$
proves $h (text{mod} p)$ is also reducible in $mathbb Z_p$



However, this proof tells nothing about $p$ being prime. And consider
$$ barphi: mathbb Z to mathbb Z / 4mathbb Z $$
is also a homomorphism, defined by
$$ z mapsto z + 4mathbb Z$$







abstract-algebra






share|cite|improve this question















share|cite|improve this question













share|cite|improve this question




share|cite|improve this question








edited Dec 10 '18 at 13:54







Astrick Harren

















asked Dec 10 '18 at 13:42









Astrick HarrenAstrick Harren

614




614












  • $begingroup$
    $2x^2+4$ is irreducible over $mathbb{Z}$, but over $mathbb{Z}_4$ it splits as $(2x)x$.
    $endgroup$
    – Randall
    Dec 10 '18 at 13:47










  • $begingroup$
    You're right, there's no need for $p$ to be prime.
    $endgroup$
    – Stephen
    Dec 10 '18 at 13:56










  • $begingroup$
    @Randall That does not mean the statement is wrong, since from the statement, any polynomial reducible in $mathbb Z_4$ says nothing about its irreducibility in $mathbb Z$
    $endgroup$
    – Astrick Harren
    Dec 10 '18 at 13:57










  • $begingroup$
    @AstrickHarren good point
    $endgroup$
    – Randall
    Dec 10 '18 at 14:05


















  • $begingroup$
    $2x^2+4$ is irreducible over $mathbb{Z}$, but over $mathbb{Z}_4$ it splits as $(2x)x$.
    $endgroup$
    – Randall
    Dec 10 '18 at 13:47










  • $begingroup$
    You're right, there's no need for $p$ to be prime.
    $endgroup$
    – Stephen
    Dec 10 '18 at 13:56










  • $begingroup$
    @Randall That does not mean the statement is wrong, since from the statement, any polynomial reducible in $mathbb Z_4$ says nothing about its irreducibility in $mathbb Z$
    $endgroup$
    – Astrick Harren
    Dec 10 '18 at 13:57










  • $begingroup$
    @AstrickHarren good point
    $endgroup$
    – Randall
    Dec 10 '18 at 14:05
















$begingroup$
$2x^2+4$ is irreducible over $mathbb{Z}$, but over $mathbb{Z}_4$ it splits as $(2x)x$.
$endgroup$
– Randall
Dec 10 '18 at 13:47




$begingroup$
$2x^2+4$ is irreducible over $mathbb{Z}$, but over $mathbb{Z}_4$ it splits as $(2x)x$.
$endgroup$
– Randall
Dec 10 '18 at 13:47












$begingroup$
You're right, there's no need for $p$ to be prime.
$endgroup$
– Stephen
Dec 10 '18 at 13:56




$begingroup$
You're right, there's no need for $p$ to be prime.
$endgroup$
– Stephen
Dec 10 '18 at 13:56












$begingroup$
@Randall That does not mean the statement is wrong, since from the statement, any polynomial reducible in $mathbb Z_4$ says nothing about its irreducibility in $mathbb Z$
$endgroup$
– Astrick Harren
Dec 10 '18 at 13:57




$begingroup$
@Randall That does not mean the statement is wrong, since from the statement, any polynomial reducible in $mathbb Z_4$ says nothing about its irreducibility in $mathbb Z$
$endgroup$
– Astrick Harren
Dec 10 '18 at 13:57












$begingroup$
@AstrickHarren good point
$endgroup$
– Randall
Dec 10 '18 at 14:05




$begingroup$
@AstrickHarren good point
$endgroup$
– Randall
Dec 10 '18 at 14:05










1 Answer
1






active

oldest

votes


















3












$begingroup$

The question of reducibility in $R[X]$ is more complicated when $R$ is not a field, so we usually restrict our attention to fields.



For example, $5x+1=(2x+1)(3x+1)$ is reducible in $mathbb{Z}/6mathbb{Z}$. When $p$ is not prime, reducibility is too common to be interesting.






share|cite|improve this answer









$endgroup$













  • $begingroup$
    Does factorization in monic polynomials change something ? $mathbb{Z}/(n)[x]$ factorizes in product of $mathbb{Z}/(p_i^{k_i})[x]$. If $h$ is irreducible $bmod p^k$ is it irreducible modulo $q$ for many primes $q$ ?
    $endgroup$
    – reuns
    Dec 10 '18 at 14:02












  • $begingroup$
    @reuns Even irreducibility mod $mathbb{Z}$, which is much stronger, is not sufficient to be irreducible modulo any prime. But I think that no polynomial is irreducible modulo a compositie integer...
    $endgroup$
    – Slade
    Dec 10 '18 at 14:29










  • $begingroup$
    That's why we should restrict to monic factorization (ask that $n$ is coprime with the leading coefficient and look at $f equiv a_d gh bmod n$). If $h$ is irreducible $bmod p$ then it is irreducible modulo $q$ for many primes $q$ by Chebotarev density theorem
    $endgroup$
    – reuns
    Dec 10 '18 at 14:51












  • $begingroup$
    $x^2+1$ is irreducible modulo $4$. I think $f = (x-sqrt{2}-sqrt{3})(x+sqrt{2}-sqrt{3})(x-sqrt{2}+sqrt{3})(x+sqrt{2}+sqrt{3})$ is irreducible in $mathbb{Z}[x]$, reducible modulo every prime, and reducible in $mathbb{Q}_p$ for $p ge 5$ (as $mathbb{Q}_p(sqrt{2},sqrt{3})/mathbb{Q}_p$ is unramified). As $(2 | 3) = -1$, $mathbb{Q}_3(sqrt{2},sqrt{3})/mathbb{Q}_3$ is a tower of a non-trivial unramified and ramified extension, thus $f$ is irreducible in $mathbb{Q}_3$ whence irreducible in $mathbb{Z}/(3^k)$ for some $k$
    $endgroup$
    – reuns
    Dec 10 '18 at 15:17













Your Answer





StackExchange.ifUsing("editor", function () {
return StackExchange.using("mathjaxEditing", function () {
StackExchange.MarkdownEditor.creationCallbacks.add(function (editor, postfix) {
StackExchange.mathjaxEditing.prepareWmdForMathJax(editor, postfix, [["$", "$"], ["\\(","\\)"]]);
});
});
}, "mathjax-editing");

StackExchange.ready(function() {
var channelOptions = {
tags: "".split(" "),
id: "69"
};
initTagRenderer("".split(" "), "".split(" "), channelOptions);

StackExchange.using("externalEditor", function() {
// Have to fire editor after snippets, if snippets enabled
if (StackExchange.settings.snippets.snippetsEnabled) {
StackExchange.using("snippets", function() {
createEditor();
});
}
else {
createEditor();
}
});

function createEditor() {
StackExchange.prepareEditor({
heartbeatType: 'answer',
autoActivateHeartbeat: false,
convertImagesToLinks: true,
noModals: true,
showLowRepImageUploadWarning: true,
reputationToPostImages: 10,
bindNavPrevention: true,
postfix: "",
imageUploader: {
brandingHtml: "Powered by u003ca class="icon-imgur-white" href="https://imgur.com/"u003eu003c/au003e",
contentPolicyHtml: "User contributions licensed under u003ca href="https://creativecommons.org/licenses/by-sa/3.0/"u003ecc by-sa 3.0 with attribution requiredu003c/au003e u003ca href="https://stackoverflow.com/legal/content-policy"u003e(content policy)u003c/au003e",
allowUrls: true
},
noCode: true, onDemand: true,
discardSelector: ".discard-answer"
,immediatelyShowMarkdownHelp:true
});


}
});














draft saved

draft discarded


















StackExchange.ready(
function () {
StackExchange.openid.initPostLogin('.new-post-login', 'https%3a%2f%2fmath.stackexchange.com%2fquestions%2f3033927%2fpolynomials-are-irreducible-in-mathbbzx-if-they-are-irreducible-in-math%23new-answer', 'question_page');
}
);

Post as a guest















Required, but never shown

























1 Answer
1






active

oldest

votes








1 Answer
1






active

oldest

votes









active

oldest

votes






active

oldest

votes









3












$begingroup$

The question of reducibility in $R[X]$ is more complicated when $R$ is not a field, so we usually restrict our attention to fields.



For example, $5x+1=(2x+1)(3x+1)$ is reducible in $mathbb{Z}/6mathbb{Z}$. When $p$ is not prime, reducibility is too common to be interesting.






share|cite|improve this answer









$endgroup$













  • $begingroup$
    Does factorization in monic polynomials change something ? $mathbb{Z}/(n)[x]$ factorizes in product of $mathbb{Z}/(p_i^{k_i})[x]$. If $h$ is irreducible $bmod p^k$ is it irreducible modulo $q$ for many primes $q$ ?
    $endgroup$
    – reuns
    Dec 10 '18 at 14:02












  • $begingroup$
    @reuns Even irreducibility mod $mathbb{Z}$, which is much stronger, is not sufficient to be irreducible modulo any prime. But I think that no polynomial is irreducible modulo a compositie integer...
    $endgroup$
    – Slade
    Dec 10 '18 at 14:29










  • $begingroup$
    That's why we should restrict to monic factorization (ask that $n$ is coprime with the leading coefficient and look at $f equiv a_d gh bmod n$). If $h$ is irreducible $bmod p$ then it is irreducible modulo $q$ for many primes $q$ by Chebotarev density theorem
    $endgroup$
    – reuns
    Dec 10 '18 at 14:51












  • $begingroup$
    $x^2+1$ is irreducible modulo $4$. I think $f = (x-sqrt{2}-sqrt{3})(x+sqrt{2}-sqrt{3})(x-sqrt{2}+sqrt{3})(x+sqrt{2}+sqrt{3})$ is irreducible in $mathbb{Z}[x]$, reducible modulo every prime, and reducible in $mathbb{Q}_p$ for $p ge 5$ (as $mathbb{Q}_p(sqrt{2},sqrt{3})/mathbb{Q}_p$ is unramified). As $(2 | 3) = -1$, $mathbb{Q}_3(sqrt{2},sqrt{3})/mathbb{Q}_3$ is a tower of a non-trivial unramified and ramified extension, thus $f$ is irreducible in $mathbb{Q}_3$ whence irreducible in $mathbb{Z}/(3^k)$ for some $k$
    $endgroup$
    – reuns
    Dec 10 '18 at 15:17


















3












$begingroup$

The question of reducibility in $R[X]$ is more complicated when $R$ is not a field, so we usually restrict our attention to fields.



For example, $5x+1=(2x+1)(3x+1)$ is reducible in $mathbb{Z}/6mathbb{Z}$. When $p$ is not prime, reducibility is too common to be interesting.






share|cite|improve this answer









$endgroup$













  • $begingroup$
    Does factorization in monic polynomials change something ? $mathbb{Z}/(n)[x]$ factorizes in product of $mathbb{Z}/(p_i^{k_i})[x]$. If $h$ is irreducible $bmod p^k$ is it irreducible modulo $q$ for many primes $q$ ?
    $endgroup$
    – reuns
    Dec 10 '18 at 14:02












  • $begingroup$
    @reuns Even irreducibility mod $mathbb{Z}$, which is much stronger, is not sufficient to be irreducible modulo any prime. But I think that no polynomial is irreducible modulo a compositie integer...
    $endgroup$
    – Slade
    Dec 10 '18 at 14:29










  • $begingroup$
    That's why we should restrict to monic factorization (ask that $n$ is coprime with the leading coefficient and look at $f equiv a_d gh bmod n$). If $h$ is irreducible $bmod p$ then it is irreducible modulo $q$ for many primes $q$ by Chebotarev density theorem
    $endgroup$
    – reuns
    Dec 10 '18 at 14:51












  • $begingroup$
    $x^2+1$ is irreducible modulo $4$. I think $f = (x-sqrt{2}-sqrt{3})(x+sqrt{2}-sqrt{3})(x-sqrt{2}+sqrt{3})(x+sqrt{2}+sqrt{3})$ is irreducible in $mathbb{Z}[x]$, reducible modulo every prime, and reducible in $mathbb{Q}_p$ for $p ge 5$ (as $mathbb{Q}_p(sqrt{2},sqrt{3})/mathbb{Q}_p$ is unramified). As $(2 | 3) = -1$, $mathbb{Q}_3(sqrt{2},sqrt{3})/mathbb{Q}_3$ is a tower of a non-trivial unramified and ramified extension, thus $f$ is irreducible in $mathbb{Q}_3$ whence irreducible in $mathbb{Z}/(3^k)$ for some $k$
    $endgroup$
    – reuns
    Dec 10 '18 at 15:17
















3












3








3





$begingroup$

The question of reducibility in $R[X]$ is more complicated when $R$ is not a field, so we usually restrict our attention to fields.



For example, $5x+1=(2x+1)(3x+1)$ is reducible in $mathbb{Z}/6mathbb{Z}$. When $p$ is not prime, reducibility is too common to be interesting.






share|cite|improve this answer









$endgroup$



The question of reducibility in $R[X]$ is more complicated when $R$ is not a field, so we usually restrict our attention to fields.



For example, $5x+1=(2x+1)(3x+1)$ is reducible in $mathbb{Z}/6mathbb{Z}$. When $p$ is not prime, reducibility is too common to be interesting.







share|cite|improve this answer












share|cite|improve this answer



share|cite|improve this answer










answered Dec 10 '18 at 13:57









SladeSlade

25k12665




25k12665












  • $begingroup$
    Does factorization in monic polynomials change something ? $mathbb{Z}/(n)[x]$ factorizes in product of $mathbb{Z}/(p_i^{k_i})[x]$. If $h$ is irreducible $bmod p^k$ is it irreducible modulo $q$ for many primes $q$ ?
    $endgroup$
    – reuns
    Dec 10 '18 at 14:02












  • $begingroup$
    @reuns Even irreducibility mod $mathbb{Z}$, which is much stronger, is not sufficient to be irreducible modulo any prime. But I think that no polynomial is irreducible modulo a compositie integer...
    $endgroup$
    – Slade
    Dec 10 '18 at 14:29










  • $begingroup$
    That's why we should restrict to monic factorization (ask that $n$ is coprime with the leading coefficient and look at $f equiv a_d gh bmod n$). If $h$ is irreducible $bmod p$ then it is irreducible modulo $q$ for many primes $q$ by Chebotarev density theorem
    $endgroup$
    – reuns
    Dec 10 '18 at 14:51












  • $begingroup$
    $x^2+1$ is irreducible modulo $4$. I think $f = (x-sqrt{2}-sqrt{3})(x+sqrt{2}-sqrt{3})(x-sqrt{2}+sqrt{3})(x+sqrt{2}+sqrt{3})$ is irreducible in $mathbb{Z}[x]$, reducible modulo every prime, and reducible in $mathbb{Q}_p$ for $p ge 5$ (as $mathbb{Q}_p(sqrt{2},sqrt{3})/mathbb{Q}_p$ is unramified). As $(2 | 3) = -1$, $mathbb{Q}_3(sqrt{2},sqrt{3})/mathbb{Q}_3$ is a tower of a non-trivial unramified and ramified extension, thus $f$ is irreducible in $mathbb{Q}_3$ whence irreducible in $mathbb{Z}/(3^k)$ for some $k$
    $endgroup$
    – reuns
    Dec 10 '18 at 15:17




















  • $begingroup$
    Does factorization in monic polynomials change something ? $mathbb{Z}/(n)[x]$ factorizes in product of $mathbb{Z}/(p_i^{k_i})[x]$. If $h$ is irreducible $bmod p^k$ is it irreducible modulo $q$ for many primes $q$ ?
    $endgroup$
    – reuns
    Dec 10 '18 at 14:02












  • $begingroup$
    @reuns Even irreducibility mod $mathbb{Z}$, which is much stronger, is not sufficient to be irreducible modulo any prime. But I think that no polynomial is irreducible modulo a compositie integer...
    $endgroup$
    – Slade
    Dec 10 '18 at 14:29










  • $begingroup$
    That's why we should restrict to monic factorization (ask that $n$ is coprime with the leading coefficient and look at $f equiv a_d gh bmod n$). If $h$ is irreducible $bmod p$ then it is irreducible modulo $q$ for many primes $q$ by Chebotarev density theorem
    $endgroup$
    – reuns
    Dec 10 '18 at 14:51












  • $begingroup$
    $x^2+1$ is irreducible modulo $4$. I think $f = (x-sqrt{2}-sqrt{3})(x+sqrt{2}-sqrt{3})(x-sqrt{2}+sqrt{3})(x+sqrt{2}+sqrt{3})$ is irreducible in $mathbb{Z}[x]$, reducible modulo every prime, and reducible in $mathbb{Q}_p$ for $p ge 5$ (as $mathbb{Q}_p(sqrt{2},sqrt{3})/mathbb{Q}_p$ is unramified). As $(2 | 3) = -1$, $mathbb{Q}_3(sqrt{2},sqrt{3})/mathbb{Q}_3$ is a tower of a non-trivial unramified and ramified extension, thus $f$ is irreducible in $mathbb{Q}_3$ whence irreducible in $mathbb{Z}/(3^k)$ for some $k$
    $endgroup$
    – reuns
    Dec 10 '18 at 15:17


















$begingroup$
Does factorization in monic polynomials change something ? $mathbb{Z}/(n)[x]$ factorizes in product of $mathbb{Z}/(p_i^{k_i})[x]$. If $h$ is irreducible $bmod p^k$ is it irreducible modulo $q$ for many primes $q$ ?
$endgroup$
– reuns
Dec 10 '18 at 14:02






$begingroup$
Does factorization in monic polynomials change something ? $mathbb{Z}/(n)[x]$ factorizes in product of $mathbb{Z}/(p_i^{k_i})[x]$. If $h$ is irreducible $bmod p^k$ is it irreducible modulo $q$ for many primes $q$ ?
$endgroup$
– reuns
Dec 10 '18 at 14:02














$begingroup$
@reuns Even irreducibility mod $mathbb{Z}$, which is much stronger, is not sufficient to be irreducible modulo any prime. But I think that no polynomial is irreducible modulo a compositie integer...
$endgroup$
– Slade
Dec 10 '18 at 14:29




$begingroup$
@reuns Even irreducibility mod $mathbb{Z}$, which is much stronger, is not sufficient to be irreducible modulo any prime. But I think that no polynomial is irreducible modulo a compositie integer...
$endgroup$
– Slade
Dec 10 '18 at 14:29












$begingroup$
That's why we should restrict to monic factorization (ask that $n$ is coprime with the leading coefficient and look at $f equiv a_d gh bmod n$). If $h$ is irreducible $bmod p$ then it is irreducible modulo $q$ for many primes $q$ by Chebotarev density theorem
$endgroup$
– reuns
Dec 10 '18 at 14:51






$begingroup$
That's why we should restrict to monic factorization (ask that $n$ is coprime with the leading coefficient and look at $f equiv a_d gh bmod n$). If $h$ is irreducible $bmod p$ then it is irreducible modulo $q$ for many primes $q$ by Chebotarev density theorem
$endgroup$
– reuns
Dec 10 '18 at 14:51














$begingroup$
$x^2+1$ is irreducible modulo $4$. I think $f = (x-sqrt{2}-sqrt{3})(x+sqrt{2}-sqrt{3})(x-sqrt{2}+sqrt{3})(x+sqrt{2}+sqrt{3})$ is irreducible in $mathbb{Z}[x]$, reducible modulo every prime, and reducible in $mathbb{Q}_p$ for $p ge 5$ (as $mathbb{Q}_p(sqrt{2},sqrt{3})/mathbb{Q}_p$ is unramified). As $(2 | 3) = -1$, $mathbb{Q}_3(sqrt{2},sqrt{3})/mathbb{Q}_3$ is a tower of a non-trivial unramified and ramified extension, thus $f$ is irreducible in $mathbb{Q}_3$ whence irreducible in $mathbb{Z}/(3^k)$ for some $k$
$endgroup$
– reuns
Dec 10 '18 at 15:17






$begingroup$
$x^2+1$ is irreducible modulo $4$. I think $f = (x-sqrt{2}-sqrt{3})(x+sqrt{2}-sqrt{3})(x-sqrt{2}+sqrt{3})(x+sqrt{2}+sqrt{3})$ is irreducible in $mathbb{Z}[x]$, reducible modulo every prime, and reducible in $mathbb{Q}_p$ for $p ge 5$ (as $mathbb{Q}_p(sqrt{2},sqrt{3})/mathbb{Q}_p$ is unramified). As $(2 | 3) = -1$, $mathbb{Q}_3(sqrt{2},sqrt{3})/mathbb{Q}_3$ is a tower of a non-trivial unramified and ramified extension, thus $f$ is irreducible in $mathbb{Q}_3$ whence irreducible in $mathbb{Z}/(3^k)$ for some $k$
$endgroup$
– reuns
Dec 10 '18 at 15:17




















draft saved

draft discarded




















































Thanks for contributing an answer to Mathematics Stack Exchange!


  • Please be sure to answer the question. Provide details and share your research!

But avoid



  • Asking for help, clarification, or responding to other answers.

  • Making statements based on opinion; back them up with references or personal experience.


Use MathJax to format equations. MathJax reference.


To learn more, see our tips on writing great answers.




draft saved


draft discarded














StackExchange.ready(
function () {
StackExchange.openid.initPostLogin('.new-post-login', 'https%3a%2f%2fmath.stackexchange.com%2fquestions%2f3033927%2fpolynomials-are-irreducible-in-mathbbzx-if-they-are-irreducible-in-math%23new-answer', 'question_page');
}
);

Post as a guest















Required, but never shown





















































Required, but never shown














Required, but never shown












Required, but never shown







Required, but never shown

































Required, but never shown














Required, but never shown












Required, but never shown







Required, but never shown







Popular posts from this blog

Basket-ball féminin

Different font size/position of beamer's navigation symbols template's content depending on regular/plain...

I want to find a topological embedding $f : X rightarrow Y$ and $g: Y rightarrow X$, yet $X$ is not...